Answer to Question #41568 in Microeconomics for risismac

Question #41568
Assume the following unit cost data are for a (perfectly) competitive producer.

(b) At a product price of RM32, will this firm produce in the short run? Why, or why not? If it does produce, what will be the profit-maximising or loss-minimising output? What total economic profit or loss will the firm realise to produce at such level of output?
1
Expert's answer
2014-04-22T15:42:15-0400
Unfortunately, the statement of your question is a bit confusing.
Data in the statement is missing. Please, give us the
details so we could help you.

There is not enough information to calculate profit-maximizing output and total profit

Need a fast expert's response?

Submit order

and get a quick answer at the best price

for any assignment or question with DETAILED EXPLANATIONS!

Comments

No comments. Be the first!

Leave a comment

LATEST TUTORIALS
New on Blog
APPROVED BY CLIENTS